Difference between revisions of "1979 AHSME Problems/Problem 23"

m (Solution)
(Solution)
Line 41: Line 41:
  
 
==Solution==
 
==Solution==
 +
Note that the distance <math>PQ</math> will be minimized when <math>P</math> is the midpoint of <math>AB</math> and <math>Q</math> is the midpoint of <math>CD</math>.
 +
 +
To find this distance, consider triangle <math>\triangle PCQ</math>. <math>Q</math> is the midpoint of <math>CD</math>, so <math>CQ=\frac{1}{2}</math>. Additionally, since <math>CP</math> is the altitude of equilateral <math>\triangle ABC</math>, <math>CP=\frac{\sqrt{3}}{2}</math>.
 +
 +
Next, we need to find <math>\cos(\angle PCD)</math> in order to find <math>PQ</math> by the Law of Cosines. To do so, drop down <math>D</math> onto <math>\triangle ABC</math> to get the point <math>D^\prime</math>. Note that <math>\triangle CD^\prime D</math> is a right triangle with <math>\angle CD^\prime D</math> as a right angle. As given by the problem, <math>CD=1</math>.
 +
 +
 +
Note that <math>D^\prime</math> is the centroid of equilateral <math>\triangle ABC</math>. Additionally, since <math>\triangle ABC</math> is equilateral, <math>D^\prime</math> is also the orthocenter. Due to this, the distance from <math>C</math> to <math>D^\prime</math> is <math>\frac{2}{3}</math> of the altitude of <math>\triangle ABC</math>. Therefore, <math>CD^\prime=\frac{\sqrt{3}}{3}</math>.
 +
 +
Since <math>\cos(\angle PCD)=\frac{CD^\prime}{CD}</math>, <math>\cos(\angle PCD)=\frac{\frac{\sqrt{3}}{3}}{1}=\frac{\sqrt{3}}{3}</math>
 +
 +
<cmath>PQ^2=CP^2+CQ^2-2(CP)(CQ)\cos(\angle PCD)</cmath>
 +
<cmath>PQ^2=\frac{3}{4}+\frac{1}{4}-2\left(\frac{\sqrt{3}}{4}\right)\left(\frac{1}{2}\right)\left(\frac{\sqrt{3}}{3}\right)</cmath>
 +
Simplifying, <math>PQ^2=\frac{1}{2}</math>.
 +
Therefore, <math>PQ=\frac{\sqrt{2}}{2}</math>
 +
 
<math>\boxed{\textbf{C}}</math>
 
<math>\boxed{\textbf{C}}</math>
  

Revision as of 10:23, 12 February 2018

Problem 23

The edges of a regular tetrahedron with vertices $A ,~ B,~ C$, and $D$ each have length one. Find the least possible distance between a pair of points $P$ and $Q$, where $P$ is on edge $AB$ and $Q$ is on edge $CD$.


[asy] size(150); import patterns; pair D=(0,0),C=(1,-1),B=(2.5,-0.2),A=(1,2),AA,BB,CC,DD,P,Q,aux; add("hatch",hatch()); //AA=new A and etc. draw(rotate(100,D)*(A--B--C--D--cycle)); AA=rotate(100,D)*A; BB=rotate(100,D)*D; CC=rotate(100,D)*C; DD=rotate(100,D)*B; aux=midpoint(AA--BB); draw(BB--DD); P=midpoint(AA--aux); aux=midpoint(CC--DD); Q=midpoint(CC--aux); draw(AA--CC,dashed); dot(P); dot(Q); fill(DD--BB--CC--cycle,pattern("hatch")); label("$A$",AA,W); label("$B$",BB,S); label("$C$",CC,E); label("$D$",DD,N); label("$P$",P,S); label("$Q$",Q,E); //Credit to TheMaskedMagician for the diagram[/asy]


$\textbf{(A) }\frac{1}{2}\qquad \textbf{(B) }\frac{3}{4}\qquad \textbf{(C) }\frac{\sqrt{2}}{2}\qquad \textbf{(D) }\frac{\sqrt{3}}{2}\qquad \textbf{(E) }\frac{\sqrt{3}}{3}$

Solution

Note that the distance $PQ$ will be minimized when $P$ is the midpoint of $AB$ and $Q$ is the midpoint of $CD$.

To find this distance, consider triangle $\triangle PCQ$. $Q$ is the midpoint of $CD$, so $CQ=\frac{1}{2}$. Additionally, since $CP$ is the altitude of equilateral $\triangle ABC$, $CP=\frac{\sqrt{3}}{2}$.

Next, we need to find $\cos(\angle PCD)$ in order to find $PQ$ by the Law of Cosines. To do so, drop down $D$ onto $\triangle ABC$ to get the point $D^\prime$. Note that $\triangle CD^\prime D$ is a right triangle with $\angle CD^\prime D$ as a right angle. As given by the problem, $CD=1$.


Note that $D^\prime$ is the centroid of equilateral $\triangle ABC$. Additionally, since $\triangle ABC$ is equilateral, $D^\prime$ is also the orthocenter. Due to this, the distance from $C$ to $D^\prime$ is $\frac{2}{3}$ of the altitude of $\triangle ABC$. Therefore, $CD^\prime=\frac{\sqrt{3}}{3}$.

Since $\cos(\angle PCD)=\frac{CD^\prime}{CD}$, $\cos(\angle PCD)=\frac{\frac{\sqrt{3}}{3}}{1}=\frac{\sqrt{3}}{3}$

\[PQ^2=CP^2+CQ^2-2(CP)(CQ)\cos(\angle PCD)\] \[PQ^2=\frac{3}{4}+\frac{1}{4}-2\left(\frac{\sqrt{3}}{4}\right)\left(\frac{1}{2}\right)\left(\frac{\sqrt{3}}{3}\right)\] Simplifying, $PQ^2=\frac{1}{2}$. Therefore, $PQ=\frac{\sqrt{2}}{2}$

$\boxed{\textbf{C}}$

See also

1979 AHSME (ProblemsAnswer KeyResources)
Preceded by
Problem 22
Followed by
Problem 24
1 2 3 4 5 6 7 8 9 10 11 12 13 14 15 16 17 18 19 20 21 22 23 24 25 26 27 28 29 30
All AHSME Problems and Solutions

The problems on this page are copyrighted by the Mathematical Association of America's American Mathematics Competitions. AMC logo.png